[obm-l] Re: [obm-l] Probabilidade de que o número de sucessos seja par

2015-10-12 Por tôpico Lucas Prado Melo
Correção: a recorrência é Pn = p (1-P(n-1)) + (1-p) P(n-1)

2015-10-12 21:42 GMT-03:00 Lucas Prado Melo <luca...@dcc.ufba.br>:

> É possível mostrar que Pn = p *( 1-  P(n-1)) + (1-p) Pn
>
> Disso conclui-se que Pn = p + (1-2p)P(n-1)   e, dividindo a equação por
> (1-2p)^n (para p != 1/2), encontramos uma formula fechada para Pn/(1-2p)^n.
>
> Finalmente chegamos que Pn = (1 + (1-2p)^n)/2, mesmo quando p = 1/2.
>
> 2015-10-12 20:17 GMT-03:00 Amanda Merryl <sc...@hotmail.com>:
>
>>
>> Oi amigos
>>
>> Um experimento tem probabilidade p de sucesso. Em n realizaçŠes
>> independentes do mesmo, qual a probabilidade Pn de que o número de
>> sucessos seja par? Há uma fórmula  fechada para Pn?
>>
>> Devemos ter lim n --> oo Pn = 1/2, certo?
>>
>> Obrigada.
>>
>> Amanda
>>
>>
>> --
>> Esta mensagem foi verificada pelo sistema de antivírus e
>>  acredita-se estar livre de perigo.
>>
>>
>> =
>> Instruções para entrar na lista, sair da lista e usar a lista em
>> http://www.mat.puc-rio.br/~obmlistas/obm-l.html
>> =
>>
>
>
>
> --
> []'s
> Lucas
>



-- 
[]'s
Lucas

-- 
Esta mensagem foi verificada pelo sistema de antiv�rus e
 acredita-se estar livre de perigo.



[obm-l] Re: [obm-l] Probabilidade de que o número de sucessos seja par

2015-10-12 Por tôpico Lucas Prado Melo
É possível mostrar que Pn = p *( 1-  P(n-1)) + (1-p) Pn

Disso conclui-se que Pn = p + (1-2p)P(n-1)   e, dividindo a equação por
(1-2p)^n (para p != 1/2), encontramos uma formula fechada para Pn/(1-2p)^n.

Finalmente chegamos que Pn = (1 + (1-2p)^n)/2, mesmo quando p = 1/2.

2015-10-12 20:17 GMT-03:00 Amanda Merryl :

>
> Oi amigos
>
> Um experimento tem probabilidade p de sucesso. Em n realizaçŠes
> independentes do mesmo, qual a probabilidade Pn de que o número de
> sucessos seja par? Há uma fórmula  fechada para Pn?
>
> Devemos ter lim n --> oo Pn = 1/2, certo?
>
> Obrigada.
>
> Amanda
>
>
> --
> Esta mensagem foi verificada pelo sistema de antivírus e
>  acredita-se estar livre de perigo.
>
>
> =
> Instruções para entrar na lista, sair da lista e usar a lista em
> http://www.mat.puc-rio.br/~obmlistas/obm-l.html
> =
>



-- 
[]'s
Lucas

-- 
Esta mensagem foi verificada pelo sistema de antiv�rus e
 acredita-se estar livre de perigo.



[obm-l] Re: [obm-l] Uma sequência

2015-03-31 Por tôpico Lucas Prado Melo
Indução?

2015-03-31 9:22 GMT-03:00 marcone augusto araújo borges 
marconeborge...@hotmail.com:

 Considere uma sequência an definida como

 a1 = 2:
 a(n+1) = a1.a2an + 1,(n  = 1)

 Mostre que  1/a1 + 1/a2 + ... + 1/an = 1 - 1/(a1.a2...an)

 Uma dica?

 --
 Esta mensagem foi verificada pelo sistema de antivírus e
 acredita-se estar livre de perigo.




-- 
[]'s
Lucas

-- 
Esta mensagem foi verificada pelo sistema de antiv�rus e
 acredita-se estar livre de perigo.



[obm-l] Re: [obm-l] Piso de um número real

2014-02-15 Por tôpico Lucas Prado Melo
2014-02-15 10:20 GMT-02:00 marcone augusto araújo borges 
marconeborge...@hotmail.com:

 Se x é um numero real,seja [x] o maior inteiro n tal que n  = x
 Exemplos [pi] =  3 e [3] = 3
 Seja x - [x] = ´´parte decimal de x´´
 Eu desconfio que as ´´partes decimais´´ de (n.2^1/2)/2 e n.{1 - (2^1/2)/2}
 somam 1.
 Nao consigo justificar.Alguem ajuda?

 No capítulo 3 do Concrete Mathematics, tem vários truques legais pra
trabalhar com isso.

Para n par, sua conjectura é verdadeira (eu uso * p/ multiplicação):

(n * 2^1/2)/2 + n * (1 - 2^1/2)/2 - [(n * 2^1/2)/2] - n * (1 - 2^1/2)/2
= n/2 - [(n * 2^1/2)/ 2 + [n(1 - 2^1/2) / 2] ](a gente pode por
inteiros dentro)
= n/2 - [(n * 2^1/2)/2 + [n/2 - n*2^1/2 / 2] ]  (n/2 é inteiro e sai)
= - [(n * 2^1/2)/2 + [- n * (2^1/2) / 2 ]
= - [(n * 2^1/2)/2 - ]n * (2^1/2) / 2[ ] (aqui usamos a função ]x[ =
min{n inteiro | n = x} e a propriedade [-x] = -]x[... a notação ficou um
pouco confusa...)
Como (n*2^1/2)/2 é irracional nós obtemos na expressão acima: - [ -0.abc...
]  onde 0.abc é uma fração maior que 0. E isso se reduz ao valor 1.

Fazendo operações similares pra n ímpar concluímos que:

-1/2 - [ (n * 2^1/2)/2 - ](n * 2^1/2 + 1)/2[ ]  (aqui eu usei o fato de que
(n+1)/2 é inteiro e compensei de acordo)
Esse valor não pode ser inteiro já que a expressão entre [ ] vai resultar
em um número inteiro que será subtraído de -1/2.


-- 
[]'s
Lucas

-- 
Esta mensagem foi verificada pelo sistema de antivírus e
 acredita-se estar livre de perigo.



[obm-l] Re: [obm-l] Re: [obm-l] Análise combinatória

2013-07-12 Por tôpico Lucas Prado Melo
2013/7/12 Marcos Martinelli mffmartine...@gmail.com

 Seja {A_n} a quantidade de seqüências com 4 números escolhidos de 1 a n
 tais que a diferença positiva seja maior ou igual a 2 (n=4).

 Seja {B_n} a quantidade de seqüências com 3 números escolhidos de 1 a n
 tais que a diferença positiva seja maior ou igual a 2 (n=3).

 Seja {C_n} a quantidade de seqüências com 2 números escolhidos de 1 a n
 tais que a diferença positiva seja maior ou igual a 2 (n=2).

 Para sabermos quanto vale A_(n+1), devemos dividir nossa contagem em duas
 partes:

 i) escolher 4 números dentre os que vão de 1 a n tais que a diferença
 positiva seja maior ou igual a 2. Isto pode ser feito de A_n maneiras.

 ii) escolher o (n+1) como um número obrigatório a constar no nosso
 conjunto de 4. Após isso, escolher 3 números entre os que vão de 1 a (n-1),
 cuja diferença positiva seja maior ou igual a 2. Isto pode ser feito de
 B_(n-1) maneiras.

 Podemos escrever: A_(n+1) = A_n + B_(n-1) (n=4).

 Analogamente teremos: B_(n+1) = B_n + C_(n-1) (n=3).

 Pensando de maneira similar, temos também: C_(n+1) = C_n + (n-1) (n=2).

 Temos três séries telescópicas. Resolvendo e lembrando que a soma das
 colunas do triângulo de Pascal é o número binomial localizado na diagonal à
 direita do último elemento do somatório, obteremos:

 C_n = binomial (n-1,2) = (n-1).(n-2)/2!

 B_n = binomial (n-2,3) = (n-2)(n-3)(n-4)/3!

 A_n = binomial (n-3,4) = (n-3)(n-4)(n-5)(n-6)/4!


Interessante a solução, ela me faz pensar o seguinte:
há uma bijeção entre uma escolha (x1, x2, x3, x4) em números de 1 a n com a
restrição, e uma escolha (x1, x2-1, x3-2, x4-3) para números de 1 a n-3 sem
a restrição. Como este último pode ser escolhido de binomial(n-3, 4)
formas, então o primeiro também poderia.

-- 
[]'s
Lucas

-- 
Esta mensagem foi verificada pelo sistema de antivírus e
 acredita-se estar livre de perigo.



[obm-l] Re: [obm-l] Re: [obm-l] Re: [obm-l] Re: [obm-l] Análise combinatória

2013-07-12 Por tôpico Lucas Prado Melo
2013/7/12 Marcos Martinelli mffmartine...@gmail.com

 Mas vc conseguiu mostrar que existe mesmo a bijeção?


Um representante do primeiro tera um único representante no segundo e
vice-versa pois só é feita uma subtração/soma.

A questão é somente se as restrições são respeitadas.

x2-1  x1 sse x2-x1 = 2
x3-2  x2-1 sse x3-x2 = 2
x4-3  x3-2 sse x4-x3 = 2

x4 = n sse x4-3 = n-3
x1 = 1 sse x1 = 1

-- 
[]'s
Lucas

-- 
Esta mensagem foi verificada pelo sistema de antivírus e
 acredita-se estar livre de perigo.



[obm-l] Re: [obm-l] Divisibilidade(congruência)

2013-07-11 Por tôpico Lucas Prado Melo
2013/7/11 Artur Costa Steiner steinerar...@gmail.com

 O Bernardo já mostrou que m + n é múltiplo de 3. Resta mostrar que é
 também múltiplo de 8. Pelo mesmo raciocínio, mn = -1 (mod 8). Para que isto
 seja possível, um dos números m e n tem que ser congruente a 1 módulo 8 e,
 o outro, congruente a -1. Logo, m + n = 1 + (-1) = 0 (mod 8), ou seja,  m +
 n é múltiplo de 8

 m poderia ser 3 e n ser 5.

3*5 = 15 = 16 - 1 = -1 (mod 8)

-- 
[]'s
Lucas

-- 
Esta mensagem foi verificada pelo sistema de antivírus e
 acredita-se estar livre de perigo.



[obm-l] Re: [obm-l] Análise combinatória

2013-07-11 Por tôpico Lucas Prado Melo
2013/7/11 Artur Costa Steiner steinerar...@gmail.com

 Não consegui achar uma forma de resolver isto sem recorrer a um
 computador.

 Com os inteiros de 1 a 100, quantos conjuntos de 4 elementos podemos
 formar de modo que a diferença positiva entre dois elementos do conjunto
 seja maior ou igual a 2?


Utilizando da seguinte identidade:

sum_{0 = k = n}  kCm = (n+1)C(m+1)

, onde xCy é o numero de combinações de x objetos, y a y,

podemos obter uma expressão para o valor procurado.

Em vez de considerar as posições, consideremos a posição inicial x e as
diferenças d1, d2 e d3, de modo que os números selecionados sejam x, x+d1,
x+d1+d2, x+d1+d2+d3.

Se fixarmos k = d1+d2+d3, de quantas formas podemos selecionar uma tripla
(d1, d2, d3)? Basta fazer uma combinação completa de  k-6 em 3 pedaços e
então adicionar a cada um dos 3 pedaços o +2 pra respeitar a restrição de
ser = 2. O número de triplas é portanto (k-6 + 2)C2.

Fixado k, podemos selecionar a posição inicial de 100-k-1 formas.

Assim o número total de formas é
sum_{6 = k = 99} (100 - k - 1) (k-6 +2)C2.

Para nos livrarmos do k no fator podemos fazer o seguinte:

(99 - 3 - (k-3)) (k-4)C2 = 96 (k-4)C2   -(k-3) (k-4)C2  = 96 (k-4)C2
-   3  (k-3)C3

E assim a expressão pode ser obtida da identidade mostrada no início com
algumas manipulaçõezinhas algébricas.


-- 
[]'s
Lucas

-- 
Esta mensagem foi verificada pelo sistema de antivírus e
 acredita-se estar livre de perigo.



[obm-l] Re: [obm-l] Re: [obm-l] Re: [obm-l] Re: [obm-l] questão bacana(quase me tira o sono)

2013-06-13 Por tôpico Lucas Prado Melo
2013/6/13 Cassio Anderson Feitosa cassiofeito...@gmail.com

 Eu pensei também no problema e vou mostrar o que pensei pra que possam me
 mostrar o erro, se houver.

  Como 2^0+2^1 +  . . . + 2^{99}  =  2^{100} -1  2^{100}, então não
 importa a forma que distribuímos os pesos, o prato com 2^{100} gramas
 sempre será mais pesado. Então, o peso com 2^{100} gramas deverá ficar no
 prato direito. Tendo isto, não importa como distribuímos os outros 100
 pesos, o prato esquerdo nunca será mais pesado que o direito.

  Daí pensei de quantas formas podemos distribuir esses 100 pesos.
 Interpretei que cada escolha de pesos pro prato direito seria equivalente a
 um subconjunto de {2^0, 2^1, . . .,  2^{99}}, onde concluí que teria
 2^{100} maneiras de distribuir esses pesos entre os pratos (considerando
 que um prato pode ficar vazio).

  Foi isso o que pensei.


Acho que o problema aí está na ordem em que vc coloca os pratos. Não
necessariamente o prato 2^100 iria ser colocado no início, então os outros
pratos não teriam essa liberdade.

Num caso menor, com 1, 2, 4, existem as seguintes possibilidades:

1 (d), 2 (d), 4(d)
1 (d), 4 (d), 2(e)
1 (d), 4 (d), 2(d)
2 (d), 1 (e), 4(d)
2 (d), 1 (d), 4(d)
2 (d), 4(d), 1(e)
2 (d), 4(d), 1(d)
4 (d), 1(e), 2(e)
4 (d), 1(e), 2(d)
4 (d), 1(d), 2(e)
4 (d), 1(d), 2(d)
4 (d), 2(e), 1(e)
4 (d), 2(e), 1(d)
4 (d), 2(d), 1(e)
4 (d), 2(d), 1(d)


-- 
[]'s
Lucas

-- 
Esta mensagem foi verificada pelo sistema de antivírus e
 acredita-se estar livre de perigo.



[obm-l] Re: [obm-l] RE: [obm-l] Re: [obm-l] Re: [obm-l] questão bacana(quase me tira o sono)

2013-06-13 Por tôpico Lucas Prado Melo
2013/6/13 marcone augusto araújo borges marconeborge...@hotmail.com

 Olá,Lucas
 Não entendi bem a passagem ´´...a colocação das i-1 bolinhas menores não
 afetariam em nada o cálculo...´´

Ok eu viajei um pouco nesse trecho.

Eu quis dizer que as i-1 bolinhas poderiam ser colocadas livremente. Não
importa mais se elas vão pra direita ou esquerda.


 O que significa ´´escaladas de 2n´´?

Escaladas de 2n significa multiplicada por 2n.


 Vc poderia detalhar um pouco mais essa parte: F(n+1) = F(n) + 2nF(n)?


Observando o somatório, temos que F(n) está sendo somado por vários termos
na forma  g(n) F(i)/(i! 2^i) onde f é uma função.
Quando observamos o mesmo para F(n+1) os termos com fatores F(i)/(i! 2^i)
ainda aparecem, mas o coeficiente muda: g(n+1) = 2n g(n).

A princípio poderiamos então pensar que F(n+1) = 2n F(n), mas isso não é
verdade, pq quando passamos de F(n) para F(n+1) surge um novo termo no
somatório cujo fator é F(n)/(n! 2^n) que é o termo para quando i=n. Assim
F(n+1) = F(n) + 2nF(n)


-- 
[]'s
Lucas

-- 
Esta mensagem foi verificada pelo sistema de antivírus e
 acredita-se estar livre de perigo.



[obm-l] Re: [obm-l] RE: [obm-l] Re: [obm-l] Re: [obm-l] questão bacana(quase me tira o sono)

2013-06-13 Por tôpico Lucas Prado Melo
2013/6/13 Lucas Prado Melo luca...@dcc.ufba.br


 Observando o somatório, temos que F(n) está sendo somado por vários termos
 na forma  g(n) F(i)/(i! 2^i) onde f é uma função.
 Quando observamos o mesmo para F(n+1) os termos com fatores F(i)/(i! 2^i)
 ainda aparecem, mas o coeficiente muda: g(n+1) = 2n g(n).


(...) onde g é uma função. (...)

-- 
[]'s
Lucas

-- 
Esta mensagem foi verificada pelo sistema de antivírus e
 acredita-se estar livre de perigo.



[obm-l] Re: [obm-l] RE: [obm-l] Re: [obm-l] Re: [obm-l] questão bacana(quase me tira o sono)

2013-06-13 Por tôpico Lucas Prado Melo
2013/6/13 Lucas Prado Melo luca...@dcc.ufba.br


 Observando o somatório, temos que F(n) está sendo somado por vários termos
 na forma  g(n) F(i)/(i! 2^i) onde f é uma função.
 Quando observamos o mesmo para F(n+1) os termos com fatores F(i)/(i! 2^i)
 ainda aparecem, mas o coeficiente muda: g(n+1) = 2n g(n).


(...) F(n) está sendo obtido através da soma de vários termos(...)

Eu erro tanto...

-- 
[]'s
Lucas

-- 
Esta mensagem foi verificada pelo sistema de antivírus e
 acredita-se estar livre de perigo.



[obm-l] Re: [obm-l] Fwd: [obm-l] questão bacana(quase me tira o sono)

2013-06-13 Por tôpico Lucas Prado Melo
2013/6/13 Ralph Teixeira ralp...@gmail.com


 Que tal assim -- pense numa maneira de colocar os pesos como uma fila de
 pesos (na ordem em que eles serao colocados) E TAMBEM um bando de post-its,
 um pregado em cada peso, com as letras D ou E dizendo onde aquele peso vai.

 Entao, seja F(n) o numero de maneiras de montar uma fila com os N pesos
 2^0,...,2^(N-1) satisfazendo as condicoes do enunciado (isto eh, numa ordem
 e escolhendo os post-its de forma que, ao colocar os pesos da fila nos
 pratos o lado esquerdo nunca pese mais que o direito). Note que o numero de
 maneiras de colocar os pesos 2^1,...,2^N na maneira do enunciado tambem eh
 F(N) (afinal eu soh multipliquei todos os pesos por 2).

 Para encontrar uma maneira de colocar os pesos 2^0, 2^1,...,2^N, voce vai
 ter que fazer o seguinte:
 i) Decida a ordem e o lado onde voce vai colocar 2^1, 2^2, ..., 2^N. Ha
 F(N) maneiras de fazer isto.
 ii) Agora voce vai ter que encaixar o 2^0 em alguma posicao -- sao N+1
 posicoes, a saber antes do 1o ou entre o 1o e o 2o ou ... em ultimo.
 Mas, se voce colocar ele no inicio da fila, teria que ser com o post-it D
 (direito); em qualquer outra das N posicoes, vale D ou E. Entao sao 2N+1
 opcoes para encaixar o peso 2^0 na fila.

 Assim, para cada 1 maneira de botar os N pesos 2^1, 2^2, ..., 2^N,
 ha exatemente 2N+1 maneiras (distintas!) de botar os N+1 pesos 2^0,
 2^1,..., 2^N.

 Note que esta correspondencia eh biunivoca no seguinte sentido: se voce me
 der uma maneira de botar os N+1 pesos, eu jogo fora o peso 2^0 e sigo a sua
 ordem (e post-its) para colocar os N pesos 2^1, 2^2, ..., 2^N dum jeito
 valido!

 Entao F(N+1)=(2N+1).F(N). Como F(1)=1, vem F(2)=3, F(3)=3x5, F(4)=3x5x7,
 etc..

 Em suma, F(N)=1x3x5x7xx(2N-1) e F(101)=1x3x5x7x...x201 como o Lucas
 jah tinha dito.


Fica mais fácil de entender assim. Muito legal. :)

-- 
[]'s
Lucas

-- 
Esta mensagem foi verificada pelo sistema de antivírus e
 acredita-se estar livre de perigo.



[obm-l] Re: [obm-l] Re: [obm-l] questão bacana(quase me tira o sono)

2013-06-11 Por tôpico Lucas Prado Melo
2013/6/11 Henrique Rennó henrique.re...@gmail.com

 Acho que a solução que coloquei está errada. Pensando nos expoentes de
 forma crescente: se for apenas o peso 2^0 ele tem que estar no prato da
 direita. Acrescentando o peso 2^1, ele deve ir para o prato da direita e o
 peso anterior tem 2^1 possibilidades. Acrescentando o peso 2^2, ele deve ir
 para o prato da direita e os outros pesos tem 2^2 possibilidades.
 Acrescentando o peso 2^3, ele deve ir também para o prato da direita e
 todos os outros 3 poderiam estar em um dos dois pratos, num total de 2^3
 possibilidades. Assim, como o maior pesos é 2^100, existem 2^100
 possibilidades.

 Eu consegui um valor bem maior que 2^100 pq eu supus que é possível
colocar os números em qualquer ordem:
201 x 199 x 197 x ... x 5 x 3 x 1

Podemos reinterpretar a questão transformando em bolinhas numeradas de 1 a
n, se uma bolinha for colocada no prato direito, todas as bolinhas menores
podem ser colocadas no prato esquerdo.

Seja F(n) o número de possibilidade para n bolinhas.
Se a i-ésima bolinha for colocada primeiro, a colocação das i-1 bolinhas
menores que ela não afetam em nada o cálculo (podem ser colocadas em
quaisquer lugares) e as n-i bolinhas maiores vão ser posicionadas em F(n-i)
formas possíveis (considerando somente a posição relativa delas).

Assim F(n) = soma_{1 = i = n}  { 2^(i-1) (n-1)C(i-1) (i-1)! F(n-i) }

Onde nCk é o número de combinações de n, k a k.

A idéia aqui é, das n-1 posições restantes, reservar i-1 para as bolinhas
menores que a i-ésima e aplicar o princípio multiplicativo em duas ocasiões:
1) Permutando as i-1 bolinhas nas i-1 posições reservadas:  (i-1)!
2) Decidindo em que prato cada bolinha vai ficar: 2^(i-1)

Daí só falta escolher a posição relativa das n-i bolinhas restantes, o que
é representado pela multiplicação por F(n-i).

A expressão fica difícil assim. É melhor transformar F(n-i) em F(i), assim
substituimos i por n-i:

F(n) = soma_{1 = n - i = n} { 2^(n-i-1) (n-1)C(n-i-1) (n-i-1)! F(i) }
= soma_{0 = i  n }  {2^(n-1) (n-1)! F(i) / (2^i i!) }

O que acontece quando passamos de F(n) para F(n+1)?

Surge uma parcela F(n) que não existia antes e as outras parcelas são as
mesmas só que escaladas de 2n.

Assim F(n+1) = F(n) + 2nF(n) = (1+2n)F(n)

Assim F(1) = 1, F(2) = 1 x 3, F(3) = 1 x 3 x 5 ... F(101) = 201 x 199 x 197
x ... x 5 x 3 x 1

-- 
[]'s
Lucas

-- 
Esta mensagem foi verificada pelo sistema de antivírus e
 acredita-se estar livre de perigo.



[obm-l] Re: [obm-l] Re: [obm-l] Re: [obm-l] Re: [obm-l] Re: [obm-l] questão sobre invariantes - alguém poderia ajudar?

2013-02-25 Por tôpico Lucas Prado Melo
2013/2/25 Mauricio de Araujo mauricio.de.ara...@gmail.com

 opa, tua solução também é muito boa sem dúvida... obrigado pelo retorno,
 estava sem nenhuma ideia... citei a do Ralph apenas por uma questão de
 afinidade com o pensamento apresentado, só isso...

 De boa. :)
Tinha imaginado.

-- 
[]'s
Lucas


[obm-l] Re: [obm-l] questão sobre invariantes - alguém poderia ajudar?

2013-02-24 Por tôpico Lucas Prado Melo
2013/2/23 Mauricio de Araujo mauricio.de.ara...@gmail.com

 Os números 1, 2, ..., 20 são escritos em um quadro negro. Podemos apagar
 dois deles a e b e escrever no lugar o numero a+b+ab. Após muitas
 operações ficamos apenas com um numero.
 Qual deve ser esse numero?


O invariante vai ser a soma dos termos a_i1 * a_i2 * ... * a_ir, para cada
combinação {i1, i2, ..., ir} do conjunto {1, 2, 3, ..., n}.

Na sequência sem o 'a' e 'b' mas com a+b+ab (a sequência transformada), o
termo (a + b + ab) * x na forma acima está associado a 3 termos distintos
da sequência original: a*x, b*x e ab*x.
A volta também é verdadeira: dá pra agruparmos 3 termos da sequência
original para formarmos este termo na sequência transformada. Ou seja, a
invariante existe.

Então precisamos obter justamente esta soma.

Basta então lançarmos mão sobre a recorrência S_n = a_n*S_{n-1} + S_{n-1} =
n*S_{n-1} + S_{n-1}. Ela soma os termos com o n e sem o n.

Assim S_n = (n+1) S_{n-1}

Como S_1 = 1, S_n = (n+1)!/2.


-- 
[]'s
Lucas


[obm-l] Re: [obm-l] questão sobre invariantes - alguém poderia ajudar?

2013-02-24 Por tôpico Lucas Prado Melo
2013/2/24 Lucas Prado Melo luca...@dcc.ufba.br

 2013/2/23 Mauricio de Araujo mauricio.de.ara...@gmail.com

 Os números 1, 2, ..., 20 são escritos em um quadro negro. Podemos apagar
 dois deles a e b e escrever no lugar o numero a+b+ab. Após muitas
 operações ficamos apenas com um numero.
 Qual deve ser esse numero?


 O invariante vai ser a soma dos termos a_i1 * a_i2 * ... * a_ir, para cada
 combinação {i1, i2, ..., ir} do conjunto {1, 2, 3, ..., n}.

 Na sequência sem o 'a' e 'b' mas com a+b+ab (a sequência transformada), o
 termo (a + b + ab) * x na forma acima está associado a 3 termos distintos
 da sequência original: a*x, b*x e ab*x.
 A volta também é verdadeira: dá pra agruparmos 3 termos da sequência
 original para formarmos este termo na sequência transformada. Ou seja, a
 invariante existe.

 Então precisamos obter justamente esta soma.

 Basta então lançarmos mão sobre a recorrência S_n = a_n*S_{n-1} + S_{n-1}
 = n*S_{n-1} + S_{n-1}. Ela soma os termos com o n e sem o n.

 Assim S_n = (n+1) S_{n-1}

 Como S_1 = 1, S_n = (n+1)!/2.


Uma correção:

Na verdade a recorrência é S_n = n S_{n-1} + n + S_{n-1},

Isso dá S_n  + 1 = (n+1)(S_{n-1} + 1). Que, como Douglas mostrou, dá S_n =
(n+1)! - 1.

-- 
[]'s
Lucas


Re: [obm-l] Ajuda em um grande problema!

2013-02-24 Por tôpico Lucas Prado Melo
2013/2/24 douglas.olive...@grupoolimpo.com.br

 **

 Considere um sistema de eixos cartesianos ortogonais, e dois pontos A e B ,

 o ponto A localizado em (0,600) e o ponto B localizado em (800,0), assim

 ambos partem ao mesmo tempo e com mesmas velocidades , o ponto A

 Anda na direção NORTE-SUL( no sentido negativo de Y) e o ponto B na
 direção de A

 (seguindo o A). Pergunta-se, para um tempo muito grande o ponto B deve
 estar alinhado

 atrás de A, e quando isso acontecer , qual a distância entre eles?

O que significa estar alinhado?

-- 
[]'s
Lucas


Re: [obm-l] Ajuda em um grande problema!

2013-02-24 Por tôpico Lucas Prado Melo
2013/2/24 Ralph Teixeira ralp...@gmail.com

 Simplificacao 1: suponha que as velocidades de ambos sao 1 (se nao for,
 voce muda a escala de tempo para que sejam)

 Simplificacao 2: vou colocar o referencial em A.

 Entao A estah agora no ponto (0,0) o tempo todo. Seja (x(t),y(t)) a
 posicao de B com relacao a A. O que voce nos disse eh que a velocidade de B
 eh na direcao de A com modulo 1, isto eh:

 dx/dt=-x/(sqrt(x^2+y^2))
 dy/dt=-y/sqrt(x^2+y^2))

 Mas, pera ai, estah errado -- do lado esquerdo eu escrevi a velocidade de
 B no referencial original (e, do lado direito, eu usei o x e y do
 referencial novo)! Para consertar isto:

 dx/dt=-x/sqrt(x^2+y^2)
 dy/dt=-y/sqrt(x^2+y^2) + 1

 Isto eh um sistema de EDOs nao linear, e portanto meio chato de resolver
 na mao... Mas estah perfeito para resolver numericamente pelo PPLANE:
 http://math.rice.edu/~dfield/dfpp.html

 Vah aaquele site, coloque as EDOs na caixinha, faca o grafico na janela
 (-10,10)x(-10,10). Clique no ponto (8,-6) para ver a trajetoria de B em
 relacao a A (eu re-escalei tudo em 100 para os numeros ficarem menores,
 isto nao altera o problema), ou entao clique no menu em
 Solution/Keyboard Input of Initial Value e coloque x=8 e y=-6. A
 trajetoria de B eh uma curva que parece uma parabola (nao eh), e se
 aproxima seria de (0,2)? Por ali, nao sei. Mas note que a curva se
 aproxima desse ponto aa medida que o tempo vai para INFINITO, entao B nunca
 se alinha com A no sentido que eu entendi a sua pergunta.


Eu estou meio enferrujado em EDOs, então posso estar completamente
enganado, então eu pergunto:
Daria pra fazer y em função de x dividindo (dy/dt) por (dx/dt)?

Se der, o  Wolfram Alpha resolveu a EDO resultante fazendo y = x v(x), cuja
solução é y(x) = x sinh (c_1 + ln x)).

Eu ainda não sei exatamente o que alinhado significa, mas se der pra fazer
assim, tá aí.

-- 
[]'s
Lucas


[obm-l] Re: [obm-l] Re: [obm-l] Re: [obm-l] questão sobre invariantes - alguém poderia ajudar?

2013-02-24 Por tôpico Lucas Prado Melo
2013/2/24 Mauricio de Araujo mauricio.de.ara...@gmail.com

 Obrigado a todos pelas orientações... acredito que a ideia do Ralph está
 mais adequada por usar invariância que é o recurso solicitado na resolução.

 A minha solução não?

-- 
[]'s
Lucas


[obm-l] Re: [obm-l] Re: [obm-l] Re: [obm-l] questão sobre invariantes - alguém poderia ajudar?

2013-02-24 Por tôpico Lucas Prado Melo
2013/2/24 Lucas Prado Melo luca...@dcc.ufba.br

 2013/2/24 Mauricio de Araujo mauricio.de.ara...@gmail.com

 Obrigado a todos pelas orientações... acredito que a ideia do Ralph está
 mais adequada por usar invariância que é o recurso solicitado na resolução.

 A minha solução não?


A propósito, só pra esclarecer: eu achei a solução do Ralph ótima também
hehehe
Eu só queria chamar atenção que a minha solução também usava invariância,
pq do jeito que Maurício falou ficou parecendo que não. :)
-- 
[]'s
Lucas


[obm-l] Re: [obm-l] Sequência de Thue-Morse

2012-12-15 Por tôpico Lucas Prado Melo
2012/12/15 Lucas Prado Melo luca...@dcc.ufba.br

 O que se pode perceber dessa sequência é que a quantidade dos bits 1 da
 representação binária dos números é sempre ímpar.

 Assim se tivermos uma PA infinita, {a+ir} contida na sequência, essa
 invariante se mantem. E aí está o problema!

 Seja 2^m  a, e 2^m  r.

 Temos que a+2^m r, pertence à sequência. Como 'a' pertence à sequência
 também, o número de bits 1 de 'a' é ímpar e de 'r' é par para que a+2^m r
 tenha uma quantidade ímpar de 1s. Mas aí a+2^m r + 2^(2m) r (também da
 sequência) teria uma quantidade par de 1s, uma contradição.


Desculpe, me enganei. :(


-- 
[]'s
Lucas


[obm-l] Re: [obm-l] Sequência de Thue-Morse

2012-12-15 Por tôpico Lucas Prado Melo
2012/12/15 Pedro Angelo pedro.fon...@gmail.com

 Oi!

 Soa fácil, mas procurei na internet, tentei fazer, e não consegui de
 jeito nenhum. Alguém sabe demonstrar que a sequência de Thue-Morse não
 possui progressões aritméticas de comprimento infinito?

 Funciona assim: a sequência é gerada a partir do número 0, e aí
 fazemos negação binária (para obter 1) e concatenamos com a sequência
 acumulada (para obter 0 1). Então fazemos tudo de novo: negação (10) e
 concatena (01 10). Negação da acumulada (1001) e concatenação (0110
 1001). Negação da acumulada (10010110) e concatenação (01101001
 10010110), etc. A figurinha da wikipedia mostra direitinho como que
 faz https://en.wikipedia.org/wiki/File:Morse-Thue_sequence.gif

 Aí a gente pega a sequência:

 0 1 1 0 1 0 0 1 1 0  0  1   0   1   1   0
 0 1 2 3 4 5 6 7 8 9 10 11 12 13 14 15 (espero que fique alinhado)

 E pega a sequência dos números com 1 em cima: [1, 2, 4, 7, 8, 11, 13,
 14, ...]. Tem que provar que essa sequência não tem nenhuma progressão
 aritmética de comprimento infinito, isto é, nenhuma subsequência
 infinita da forma [a, a+n, a+2n, ...]

 alguma idéia? : )


O que se pode perceber dessa sequência é que a quantidade dos bits 1 da
representação binária dos números é sempre ímpar.

Assim se tivermos uma PA infinita, {a+ir} contida na sequência, essa
invariante se mantem. E aí está o problema!

Seja 2^m  a, e 2^m  r.

Temos que a+2^m r, pertence à sequência. Como 'a' pertence à sequência
também, o número de bits 1 de 'a' é ímpar e de 'r' é par para que a+2^m r
tenha uma quantidade ímpar de 1s. Mas aí a+2^m r + 2^(2m) r (também da
sequência) teria uma quantidade par de 1s, uma contradição.

-- 
[]'s
Lucas


[obm-l] Re: [obm-l] Sequência de Thue-Morse

2012-12-15 Por tôpico Lucas Prado Melo
2012/12/15 Lucas Prado Melo luca...@dcc.ufba.br

 2012/12/15 Pedro Angelo pedro.fon...@gmail.com

 Oi!

 Soa fácil, mas procurei na internet, tentei fazer, e não consegui de
 jeito nenhum. Alguém sabe demonstrar que a sequência de Thue-Morse não
 possui progressões aritméticas de comprimento infinito?

 Funciona assim: a sequência é gerada a partir do número 0, e aí
 fazemos negação binária (para obter 1) e concatenamos com a sequência
 acumulada (para obter 0 1). Então fazemos tudo de novo: negação (10) e
 concatena (01 10). Negação da acumulada (1001) e concatenação (0110
 1001). Negação da acumulada (10010110) e concatenação (01101001
 10010110), etc. A figurinha da wikipedia mostra direitinho como que
 faz https://en.wikipedia.org/wiki/File:Morse-Thue_sequence.gif

 Aí a gente pega a sequência:

 0 1 1 0 1 0 0 1 1 0  0  1   0   1   1   0
 0 1 2 3 4 5 6 7 8 9 10 11 12 13 14 15 (espero que fique alinhado)

 E pega a sequência dos números com 1 em cima: [1, 2, 4, 7, 8, 11, 13,
 14, ...]. Tem que provar que essa sequência não tem nenhuma progressão
 aritmética de comprimento infinito, isto é, nenhuma subsequência
 infinita da forma [a, a+n, a+2n, ...]

 alguma idéia? : )


 O que se pode perceber dessa sequência é que a quantidade dos bits 1 da
 representação binária dos números é sempre ímpar.

 Assim se tivermos uma PA infinita, {a+ir} contida na sequência, essa
 invariante se mantem. E aí está o problema!

 Seja 2^m  a, e 2^m  r.

 Temos que a+2^m r, pertence à sequência. Como 'a' pertence à sequência
 também, o número de bits 1 de 'a' é ímpar e de 'r' é par para que a+2^m r
 tenha uma quantidade ímpar de 1s. Mas aí a+2^m r + 2^(2m) r (também da
 sequência) teria uma quantidade par de 1s, uma contradição.


Pronto!

Ou 2^(2m) r - 2^m r tem quantidade ímpar de 1s, ou 2^(2m+1) r - 2^m r tem
quantidade ímpar (este último número teria 1 bit 1 a mais).
Veja:
r = 101

101
-  101

1001011

e
1010
-101
--
 10011011


-- 
[]'s
Lucas


[obm-l] Re: [obm-l] Dízima de período 9

2012-10-14 Por tôpico Lucas Prado Melo
2012/10/14 Pedro Chaves brped...@hotmail.com


 Caros Colegas:

 Pode a divisão de números naturais resultar numa dízima periódica (simples
 ou composta) de período 9?
 Como mostrar que não (ou sim) ?


Eu me lembro que meu professor uma vez mostrou um método de obter uma
dizima periódica de padrão P qualquer de tamanho n.

É uma soma do tipo

P * 10^-n + P * 10^-2n + ...

Que é uma PG infinita de soma P 10^-n / (1 - 10^-n) = P / (10^n - 1)

Ou seja, basta dividir P por um número composto de 9 noves. Ex:
0.1234123412341234... = 1234/


-- 
[]'s
Lucas


[obm-l] Re: [obm-l] Dízima de período 9

2012-10-14 Por tôpico Lucas Prado Melo
2012/10/14 Lucas Prado Melo luca...@dcc.ufba.br

 2012/10/14 Pedro Chaves brped...@hotmail.com


 Caros Colegas:

 Pode a divisão de números naturais resultar numa dízima periódica
 (simples ou composta) de período 9?
 Como mostrar que não (ou sim) ?


 Eu me lembro que meu professor uma vez mostrou um método de obter uma
 dizima periódica de padrão P qualquer de tamanho n.

 É uma soma do tipo

 P * 10^-n + P * 10^-2n + ...

 Que é uma PG infinita de soma P 10^-n / (1 - 10^-n) = P / (10^n - 1)

 Ou seja, basta dividir P por um número composto de 9 noves. Ex:
 0.1234123412341234... = 1234/


correção n noves e não 9 noves.


-- 
[]'s
Lucas


[obm-l] Re: [obm-l] Re: [obm-l] Dízima de período 9

2012-10-14 Por tôpico Lucas Prado Melo
2012/10/14 terence thirteen peterdirich...@gmail.com

 Em 14 de outubro de 2012 07:00, Pedro Chaves brped...@hotmail.com
 escreveu: Caros Colegas: Pode a divisão de números naturais resultar
 numa dízima periódica (simples ou composta) de período 9? Como mostrar que
 não (ou sim) ?
 Eu acho que não funciona - pois 0.999 ao ser convertido dá9/9. E
 basicamente, uma dizima do tipo x,y99 é a mesma coisa quexy,
 dividido por alguma potência de 10

0.9... é 1 mesmo

-- 
[]'s
Lucas


[obm-l] Re: [obm-l] Re: [obm-l] Divisão euclidiana de n por Dd (ainda não consegui)

2012-10-03 Por tôpico Lucas Prado Melo
2012/10/3 terence thirteen peterdirich...@gmail.com

 Em 3 de outubro de 2012 06:12, ennius enn...@bol.com.br escreveu:
 Caros Colegas, Gostaria de obter, se possível for, demonstração do
 teorema abaixo, em que divisão quer dizer divisão euclidiana, n é inteiro,
 D e d são inteiros positivos. Teorema:  O quociente da divisão de n por
 Dd é igual ao quociente da divisão de q por d, sendo q o quociente da
 divisão de n por D.
 O quociente de x por y é [x/y], parte inteira da divisão.
 Você quer que [n/Dd] = [[n/D]/d]
 Eu tenho boas razões para pensar que isso não é verdadeiro, pelo menosnão
 sem impor alguma restrição a D e d.


Isso é verdadeiro.
[n/Dd] = [[n/D]/d] sse d[n/Dd] = d[[n/D]/d]
(para d != 0)

d[[n/D]/d] = [n/D] - [n/D]%d, onde a%b é o resto de a na divisão por b.

De modo similar
d[n/Dd] = d[(n/D)/d] = (n/D) - (n/D)%d

Note que o resto aqui é especial aqui, pq carrega também a mantissa do
número. (n/D)%d = ([n/D] + m)%d = [n/D]%d + m, onde m é a mantissa, ou seja
a parte fracionária do número.

Assim fica claro que (n/D) - (n/D)%d = [n/D] - [n/D]%d

Isso é um exercício do Art of Computer Programming do Knuth.

-- 
[]'s
Lucas


[obm-l] Re: [obm-l] Existe uma fórmula fechada?

2012-08-02 Por tôpico Lucas Prado Melo
2012/8/1 Vanderlei * vanderma...@gmail.com

 O pipoqueiro cobra o valor de R$ 1,00 por saco de pipoca. Ele começa seu
 trabalho
 sem qualquer dinheiro para troco. Existem oito pessoas na fila do
 pipoqueiro, das quais
 quatro têm uma moeda de R$ 1,00 e quatro uma nota de R$ 2,00. Supondo uma
 arrumação aleatória para a fila formada pelas oito pessoas e que cada uma
 comprará
 exatamente um saco de pipoca, a probabilidade de que o pipoqueiro tenha
 troco para as
 quatro pessoas que pagarão com a nota de R$ 2,00 é:

 A resposta é 1/5.

 Pessoal, percebe-se claramente que para 2N pessoas, a probabilidade é
 igual a 1/(N + 1). Será que existe alguma fórmula fechada? Pensei em alguma
 recorrência ou coisa parecida.

 Obrigado,

 Vanderlei


Para contas o número de permutações válidas, tem o número catalão. [1]

[1] http://en.wikipedia.org/wiki/Catalan_number

-- 
[]'s
Lucas


[obm-l] Re: [obm-l] Re: [obm-l] combinatória

2012-05-22 Por tôpico Lucas Prado Melo
Eu calculei quantas somas existem que dá 100, retirei as somas que envolvem
2 números iguais (não existem somas com 3 números iguais que dê 100) e
então dividi por 3! para ordenar.

Para calcular quantas somas com três parcelas que existem com resultado 100
(parcelas a partir de 1), eu calculei quantas somas existem de três
parcelas para dar 97 (parcelas a partir de 0). Para isto, basta fazer
combinação de 97+2 2 a 2 (de 97+2 espaços selecione 2 marcadores, cada
parcela é representada pela quantidade de espaços em uma das partes
separadas pelos marcadores).

Para calcular a quantidade de somas com 2 números iguais, basta retirar uma
quantidade de 100 e dividir o resto por 2. Assim a quantidade retirada k
precisa ser par e precisa respeitar 0  k  100, temos então 98/2
possibilidades (lembre-se que não existem somas com 3 parcelas iguais para
o caso). É preciso multiplicar esse valor pelas permutações com repetição:
3!/2!

O resultado foi 784.
-- 
[]'s
Lucas


[obm-l] Re: [obm-l] Moldávia-2000

2011-12-10 Por tôpico Lucas Prado Melo
2011/12/10 João Maldonado joao_maldona...@hotmail.com



 106) Moldávia-2000 Para cada subconjunto não vazio X do conjunto M = {1,
 2, ..., 2000}, seja a_x a soma do menor com o maior elemento de X.
 Determine a média aritmética de todos tais números a_x assim obtidos.

 Parece que consegui uma solução de um jeito extremamente complicado de se
 generalizar (tentei fazer as contas mas devo ter errado em algo). Alguém
 tem uma outra solução para esse problema?


 Obs. O  problema está na página 42 da Eureka! 11


Uma idéia: fixe um par de números de M, multiplique sua soma pela
quantidade de subconjuntos que os tem como primeiro e último números e
então faça o mesmo para todos os outros pares e some seus resultados. Isso
dá uma expressão fechada para conjuntos M que estejam em progressão
aritmética.

-- 
[]'s
Lucas


[obm-l] Re: [obm-l] RE: [obm-l] Re: [obm-l] Moldávia-2000

2011-12-10 Por tôpico Lucas Prado Melo
2011/12/10 João Maldonado joao_maldona...@hotmail.com


 Foi exatamente o que eu fiz.

Bem, aqui está o link para a expressão que eu consegui:
http://www.wolframalpha.com/input/?i=sum_%28l%3D0%29
^%28n-2%29+%28+sum_%28i%3D1%29^%28n-l-1%29+%28+2^l++%28i+%2B+i+%2B+l+%2B+1%29%29+%29+%2B+sum_%28i%3D1%29^n+%282i%29

Você pode resolvê-la usando alguns somatórios bem-conhecidos.
(Aqui l se refere à quantidade números entre o máximo e mínimo e i é o
mínimo, a primeira soma são para mínimo e máximo distintos e a segundo para
mínimo e máximo iguais)

-- 
[]'s
Lucas


Re: [obm-l] Questao de probabilidade: o sapo e a mosca

2011-10-17 Por tôpico Lucas Prado Melo
2011/10/17 Willy George Amaral Petrenko wgapetre...@gmail.com


 Obviamente eu só vou querer usar essa estratégia se eu não sei como foram
 escolhidos os números.

 Sim, parece mágica, e não, eu não estudei em Hogwarts :)

 Nossa, isso é lindo! Será que é possível encontrar f(x) em função de
P(receber x) de modo a garantir vitória com chances maiores que 50%?
(Interessante que essa estratégia corresponda à intuição de que quanto maior
for o número, mais sensato é decidir ficar).


-- 
[]'s
Lucas


Re: [obm-l] Questao de probabilidade: o sapo e a mosca

2011-10-16 Por tôpico Lucas Prado Melo
2011/10/16 Ralph Teixeira ralp...@gmail.com

 Para mim, falta alguma especie de hipotese na distribuicao de probabilidade
 a priori dos numeros nos envelopes -- nem que seja uma chutada inventada da
 minha cabeca.

 Por outro lado, reconheco que estou pensando no problema mais simples --
 olho a distribuicao de probabilidade a priori, calculo a probabilidade do
 outro envelope ser maior que 173 (dado que este eh 173, se for o caso),
 decido. Deve haver raciocinios mais espertos que eu nao estou tentando
 fazer, ou maneiras espertas de estimar esta a priori baseado em que sabemos
 de programas de auditorio.

 (Por exemplo: sabendo o que eu sei de programas de auditorio, aposto que
 todos os numeros nos envelopes sao positivos; e nunca vi um numero
 com mais de 9 algarismos num envelope, entao eu eliminaria estes da minha
 distribuicao de probabilidade.)


Bem, o jogo se resume a escolher a alternativa que lhe trará maiores ganhos
sem que haja quaisquer razões convincentes pra escolher dentre uma e outra.
Claro que é possível fazer este tipo de análise que você está falando, mas
mesmo que exista uma regra para a geração do outro número, esta não pode ser
conhecida e esta pode ser regida por questões meta-jogo (como, por exemplo,
que a variável aleatória dará preferência a não premiação, ou a premiação,
conforme os efeitos dramáticos desejados).

Assim sendo, o melhor para vencer o jogo é escolher aleatoriamente uma das
alternativas (jogue uma moeda pra decidir) pois não há qualquer estratégia
que elimine suas chances de 50% de ganhar.

-- 
[]'s
Lucas


[obm-l] Re: [obm-l] Re: [obm-l] Inequação com resto

2010-12-15 Por tôpico Lucas Prado Melo
2010/12/14 Bernardo Freitas Paulo da Costa bernardo...@gmail.com

 2010/12/14 Lucas Prado Melo luca...@dcc.ufba.br:
  Olá,
 Oi,

  recentemente encontrei a seguinte conjectura (que ele diz parecer
 evidente
  para ele, mas que eu não consigo provar pra mim mesmo) num trabalho
  acadêmico de um colega:
  Seja a, b naturais diferentes de 0, com a = b. Seja b%a o resto de b na
  divisão por 'a'.
  Então 2*(b%a) = b
 
  Alguém poderia provar (ou dar contra-exemplo)? Eu tentei fazer uma busca
 por
  pelos 'a' e 'b' primos entre si (usando sequências de Farey), mas não
  consegui encontrar um contraxemplo com b = 1.
 Já é uma boa iniciativa (não sei porque Farey ajuda, mas você deve
 saber...) e não achar nada até 1 deveria ser um sinal bom para
 começar a procurar uma demonstração :)

 Escreva b = q*a + r (a divisão euclidiana de b por a, quociente q,
 resto r). A gente quer mostrar que 2*r = b. O que a gente sabe :
 0 = r  a
 0 = a = b, logo q = 1

 Então r = b - q*a, 2*r = r + b - q*a = b + (r - q*a). Como q = 1, q*a
 = a  r, logo o termo entre parênteses é negativo (estritamente) e
 assim 2r = b + Negativo  b.

 Veja que a idéia de provar isso foi a seguinte: fixe o a, e faça
 variar o b. Se b for muito perto do a, o resto r vai ser pequeno, e
 daí não funciona. Se b for muito maior, o resto r vai ser pequeno
 porque menor do que a. No meio do caminho, você tem b = 2a - 1, que
 deixa resto (a-1), mas, nem assim, dá certo, já que 2(a-1)  2a - 1 =
 b.


Obrigado a todos pelas respostas :-)

Eu usei Farey para encontrar pares de números primos entre si, já que quando
o par de números não é primo entre si podemos dividí-los pelo mdc e usar a
resposta do novo par para responder ao original.

Prova:
Seja o caso para ad, bd com mdc(a,b)=1.

bd = q*ad + r = d(b - aq) = r
Por definição de resto, 0=rad, então 0 = d(b-aq)  ad, e portanto 0 =
b-aq  a ...
Onde b-aq = r' que é o resto da divisão de b por a por definição.

E, no final, 2*r = db sse 2*dr' = db sse 2*r' = b

Ou seja, o teorema vale para (ad, bd) sse valer para (a, b)
-- 
[]'s
Lucas


[obm-l] Inequação com resto

2010-12-14 Por tôpico Lucas Prado Melo
Olá,

recentemente encontrei a seguinte conjectura (que ele diz parecer evidente
para ele, mas que eu não consigo provar pra mim mesmo) num trabalho
acadêmico de um colega:
Seja a, b naturais diferentes de 0, com a = b. Seja b%a o resto de b na
divisão por 'a'.
Então 2*(b%a) = b

Alguém poderia provar (ou dar contra-exemplo)? Eu tentei fazer uma busca por
pelos 'a' e 'b' primos entre si (usando sequências de Farey), mas não
consegui encontrar um contraxemplo com b = 1.

-- 
[]'s
Lucas


[obm-l] Re: [obm-l] RE: [obm-l] Re: [obm-l] RE: [obm-l] Re: [obm-l] RE: [obm-l] Limite de série

2010-11-18 Por tôpico Lucas Prado Melo
2010/11/16 Luís Lopes qed_te...@hotmail.com

  Sauda,c~oes, oi Lucas,

 Entendido. Aguardo os comentários do seu professor.


Eu falei com ele e parece que encontrar a soma da série pode envolver
conhecimentos de análise funcional (se não me engano) que estão acima da
alçada de um estudante de cálculo C. Então (acho) não poderei dar mais
detalhes sobre a solução, infelizmente. =/
(isso sugere que essa série não devia estar na lista de exercícios...)

-- 
[]'s
Lucas


[obm-l] Re: [obm-l] RE: [obm-l] Re: [obm-l] RE: [obm-l] Re: [obm-l] RE: [obm-l] Re: [obm-l] RE: [obm-l] Limite de série

2010-11-18 Por tôpico Lucas Prado Melo
2010/11/18 Luís Lopes qed_te...@hotmail.com

  Sauda,c~oes, oi Lucas,

 Gostaria de voltar ao assunto.

 Não me importarei se não entender a solução. Mas realmente
 gostaria de vê-la. Ou se não for possível (será mesmo que podemos
 calcular a soma da série??) gostaria de ter pelo menos a resposta.

 Se vc preferir, favor pedir pro seu professor me escrever diretamente.


Ele me deu a entender que não conhecia a resolução. =/

-- 
[]'s
Lucas


[obm-l] Re: [obm-l] RE: [obm-l] Re: [obm-l] RE: [obm-l] Limi te de série

2010-11-15 Por tôpico Lucas Prado Melo
2010/11/15 Luís Lopes qed_te...@hotmail.com

  Sauda,c~oes, oi Lucas,

 Troquei emails com o prof Rousseau e achar o valor da
 série dada pelo somando arctan(n)/(1+n²) está se revelando
 muito difícil. Inclusive a resposta sen 1 parece errada.

 Vc poderia nos dar alguma dica? Falar com o professor que passou
 o problema, confirmar o valor da série etc? O que foi feito em termos
 de correção e avaliação da lista?


Eu acredito que é um erro de digitação mesmo.
A lista não é rigidamente corrigida porque serve somente de suporte para a
disciplina e não como avaliação.

Como há muitos professores da disciplina, deve ser muito difícil encontrar o
autor desta questão para nos esclarecer, mas vou ver com o meu professor.

Muito obrigado por olhar a questão.

-- 
[]'s
Lucas


[obm-l] Re: [obm-l] 1 + x + x^2 + ... x^n tem no máximo dua s raízes reais

2010-11-08 Por tôpico Lucas Prado Melo
2010/11/8 Lucas Prado Melo lukepada...@gmail.com

 2010/11/6 Paulo Argolo pauloarg...@bol.com.br

 Caros amigos,

 É possível provar que a equação algébrica 1 + x + x^2 + ... x^n = 0 admite
 no máximo duas raízes reais, qualquer que seja o inteiro positivo n?

 Pela regra dos sinais de Descartes, não existe nenhuma raiz real para esta
 soma.

Ignore. Na verdade não existe raiz positiva.

-- 
[]'s
Lucas


[obm-l] Re: [obm-l] 1 + x + x^2 + ... x^n tem no máximo dua s raízes reais

2010-11-08 Por tôpico Lucas Prado Melo
2010/11/6 Paulo Argolo pauloarg...@bol.com.br

 Caros amigos,

 É possível provar que a equação algébrica 1 + x + x^2 + ... x^n = 0 admite
 no máximo duas raízes reais, qualquer que seja o inteiro positivo n?

Pela regra dos sinais de Descartes, não existe nenhuma raiz real para esta
soma.

-- 
[]'s
Lucas


[obm-l] Re: [obm-l] RE: [obm-l] Limite de série

2010-11-08 Por tôpico Lucas Prado Melo
2010/11/8 Luís Lopes qed_te...@hotmail.com

  Sauda,c~oes,
 Oi Lucas,

 Você tem a fonte deste problema?

 E favor confirmar se é mesmo arctan(n)/(1+n²). Poderia ser
 arctan [n/(1+n^2)] ?

É uma lista da disciplina de cálculo C da UFBA.
Pode ser baixada aqui: http://www.graphics.ufba.br/unid3lista2010.1.pdf
É a questão 4.m

A propósito, se vc puder responder para arctan[n/(1+n²)] acho que seria útil
também :-)

-- 
[]'s
Lucas


[obm-l] Limite de série

2010-11-03 Por tôpico Lucas Prado Melo
Olá,

como encontrar o limite da série cuja sequência é arctan(n)/(1+n²)?

-- 
[]'s
Lucas


[obm-l] Re: [obm-l] Re: [obm-l] Re: [obm-l] Funções Pares e Ímpares

2010-08-04 Por tôpico Lucas Prado Melo
2010/8/4 Henrique Rennó henrique.re...@gmail.com

 Errei colocando que h(x) = x^2 + 1/x é par. Gostaria de uma
 demonstração das seguintes propriedades:

 - A soma de duas funções de mesma paridade mantém essa paridade.
 - O produto de duas funções de mesma paridade é uma função par.
 - O produto de duas funções com paridades distintas é uma função ímpar.

 Encontrei as propriedades acima em
 http://pt.wikipedia.org/wiki/Fun%C3%A7%C3%B5es_pares_e_%C3%ADmpares.
 Não sei sei pode ser considerada uma fonte confiável.

As provas são imediatas da definição.
Uma função f é dita impar sse f(-x) = -f(x)
Uma função f é dita par sse f(-x) = f(x)

Então vamos trabalhar com os produtos. Seja g ímpar e f par:
f(-x) * g(-x) = - f(x)*g(x)
Então a função produto é ímpar.

Se ambas forem pares:
f(-x) * g(-x) = f(x) * g(x)
então o produto é par

Se ambas forem ímpares:
f(-x) * g(-x) = (-f(x))*(-g(x))
f(-x) * g(-x) = f(x) * g(x)
então o produto é par

Suponha que f é par e g é par:
Então f(-x) + g(-x) = f(x) + g(x)
Então a função da soma é par

Suponha que f é ímpar e g é impar:
Então f(-x) + g(-x) = -f(x) - g(x) = - (f(x) + g(x))
Então a soma é ímpar.

Note que criamos uma terceira função, diga h em todos os casos.
Nos casos em que trabalhamos com o produto h(x) = f(x) * g(x), e quando
trabalhamos com a soma h(x) = f(x) + g(x)
O que fizemos foi provar nos casos acima que h(-x) = h(x), para quando o h
fosse par, ou que h(-x) = -h(x) quando h fosse ímpar.

-- 
[]'s
Lucas


[obm-l] Re: [obm-l] convergência de série

2010-07-02 Por tôpico Lucas Prado Melo
2010/7/1 cleber vieira vieira_...@yahoo.com.br


 Amigos é dada a seguinte série:


 (3/4)^1 + (6/7)^2 + (9/10)^3 + ... + (3n/3n+1)^n + ...

 Gostaria de saber se ela converge ou diverge.

 obrigado
 Att
 Cleber



 Calcule o limite dos termos da série.
O limite de (3n / (3n+1))^n = (1 / (1 + 1/(3n))^n é igual a 1/e^(1/3)


-- 
[]'s
Lucas


[obm-l] Re: [obm-l] convergência de série

2010-06-27 Por tôpico Lucas Prado Melo
2010/6/27 cleber vieira vieira_...@yahoo.com.br

 Amigos é dada a seguinte série:


 1/(1*2)^1/2 + 1/(2*3)^1/2 + 1/(3*4)^1/2 + ... + 1/(n*(n+1))^1/2 + ...

 Eu tenho uma grande suspeita q posso e devo compará-la com a série 1/n^p q
 diverge para p** 1 e converge para p1 mas não estou enxergando, será q
 alguém poderia ajudar?


Essa série parece ser divergente, não?

1/(n+1) = 1/(n*(n+1))^1/2

-- 
[]'s
Lucas


Re: [obm-l] Teoria dos Grafos

2010-06-24 Por tôpico Lucas Prado Melo
2010/6/24 Johann Dirichlet peterdirich...@gmail.com

 Só me dá um pouco de teoria, ou onde eu posso achar: o que seria um heap?

Uma heap é uma árvore na qual cada vértice possui um valor numérico (este
valor numérico pode ser também chamado de chave).
A única propriedade que uma heap precisa respeitar é a seguinte: a chave de
um vértice é maior ou igual a chave de seus filhos (note que maior igual
aqui serve pra qualquer ordem linear na verdade, ou seja, poderia ser menor
ou igual também).

Geralmente as heaps são árvores binárias completas e podem ser expressas
numa sequência numérica (que simplifica a vida pra quem faz programas de
computador com elas). Seja a sequência xi. Para um vértice xi, seus filhos
são x(2*i) e x(2*i+1), o pai de um vértice xi é x(i/2) (arredondado para
baixo).

Como um checkpoint, verifique a propriedade de heap para a seguinte
sequência: {10, 5, 9, 3, 4, 7, 5} (x1 = 10)

Eu tô falando da heap binária completa pq geralmente é isso que se entende
quando se fala de heap.

-- 
[]'s
Lucas


Re: [obm-l] Teorema sobre rank de matrizes

2010-03-31 Por tôpico Lucas Prado Melo
Obrigado pelos esclarecimentos. :)
A definição do meu Cormen está correta, eu que li errado. (d'oh)
Vou tentar responder o exercício novamente.

Valeu


[obm-l] Teorema sobre rank de matrizes

2010-03-30 Por tôpico Lucas Prado Melo
Olá,

eu estava resolvendo os exercícios do livro Introdução a algoritmos de
Cormen et al. E encontrei o que eu acredito ser um erro.

No livro, a definição dita alternativa para o rank (não sei traduzir) de
uma matriz 'A' mxn é o maior valor 'r' tal que existam duas matrizes (uma
mxr e outra rxn) tais que seu produto seja igual à 'A'. (A definição
principal é a de que uma matriz tem rank 'r' se existirem no máximo 'r'
linhas/colunas linearmente independentes).

O livro também fala que para uma matriz 'A' mxn, rank(A) = min(m, n).

Então, na questão 31.1-9, é pedido pra provar que rank(AB) = min( rank(A),
rank(B) ).
No entanto, eu consegui provar que min( rank(A), rank(B) ) = rank(AB)

Este é meu argumento:
Seja 'A' uma matriz mxk
e seja 'B' uma matriz kxn

Então rank(AB) = k, já que é possível multiplicar duas matrizes mxk e kxn
pra encontrar AB, mas não se sabe se é possível encontrar duas matrizes de
maiores dimensões para se obter AB. (Ver definição acima)

Sabemos que rank(A) = min(m, k) e que rank(B) = min(k, n)
E sabemos que k = min(m, k) e que k = min(k, n)

Para a matriz A, temos:
rank(A) = min(m, k) = k = rank(AB)

Portanto, rank(A) = rank(AB). De forma análoga, rank(B) = rank(AB) e,
portanto,
rank(AB) = max( rank(A), rank(B) ) = min( rank(A), rank(B) )


Eu cometi algum engano? Se eu realmente cometi e alguém pudesse responder
este exercício pra mim, eu ficaria grato ;)


[obm-l] Re: [obm-l] RE: [obm-l] Re: [obm-l] Onde está o err o?

2010-02-02 Por tôpico Lucas Prado Melo
2010/2/2 Artur Steiner artur_stei...@hotmail.com

  Eu gostaria de frisar que, na minha opinião, o principal furo é se tentar
 provar uma hipótese partindo do princípio de que a mesma é verdadeira. Isto
 é um sofisma lógico, não pode ser empregado nem mesmo para provar o que é
 verdade. Por exemplo, se n é ímpar, então n^2 = 1 (mod 4). Isto pode ser
 provado, mas considerando-se outras propriedades dos números ímpares. Embora
 a proposição seja verdadeira, não podemos prová-la já supondo que  n^2= 1
 (mod 4). Isto, simplesmente, não é prova.
 Artur

Eu acredito que a prova por contradição está seguindo a forma correta. Se
temos que uma proposição implica a falsidade, temos (usando usando a
equivalência disto à antipositiva) que a verdade implica a negativa da
proposição.
Com símbolos (pra ficar mais claro):
x-y  sse   ¬y - ¬x
( ¬  é o sinal de negação)

x - falso  sse verdadeiro - ¬x sse ¬x

Ou seja, se a partir de x chegamos na falsidade, com certeza ¬x é
verdadeiro. Este é o modelo da prova por contradição.

A prova dada anteriormente foi:

Suponha que existe um número natural maior tal que n  1. Esta é proposição
'x'.

O maior número natural n é tal que para todo número natural m, n = m. Esta
é a definição de maior número natural.
Utilizando algumas propriedades de multiplicação em inequações (algo que
pode ser livremente suposto), temos que:
n  1 implica em n*n  n

No entanto, o 'n' é o maior número e n*n é natural, mas como n*n  n e
também n = n*n ambas as proposições chegam a uma contradição (implicam
falso).

No entanto a prova incorreta diz que o '¬x' é 'n=1', mas, na verdade, '¬x' é
'n = 1 ou não há maior número'. Neste ponto é que se usa o fato de que 'há
maior número' caindo no sofisma descrito, apesar de que esta utilização é
apenas efeito colateral de um possível engano de quem imaginou que 'x' seria
a proposição 'n1'.

Um adendo: Durante a prova por contradição não se usou o fato de que o maior
número natural é 1 para se chocar com o fato de que é também maior que 1.


[obm-l] Re: [obm-l] Onde está o erro?

2010-01-29 Por tôpico Lucas Prado Melo
2010/1/29 marcone augusto araújo borges marconeborge...@hotmail.com

  Onde está o erro na seguinte ´´prova´´ de q 1 é o maior número
 natural:´´Suponha,por absurdo,que o maior  número natural fosse um
 n1.Então,multiplicando ambos os membros desta desigualdade por n,teríamos
 (n^2)  n.Uma contradição pois estamos supondo q n é o maior número
 natural.Eu gostaria de um esclarecimento.Obrigado.

Na conclusão da 'prova' por absurdo, vc continua supondo uma falsidade:
existe um número natural que é o maior.
A prova por absurdo prova a negação da suposição inicial que foi existe um
número natural maior e este número é maior que 1. Sua negação seria algo
nestes termos: Não existe número natural maior ou o maior número é 1 (ou 0
;).


[obm-l] Re: [obm-l] Dúvidas combinatórias.

2009-09-24 Por tôpico Lucas Prado Melo
2009/9/23 Lucas Colucci lucascolu...@hotmail.com

  Olá membros da lista, gostaria de uma ajuda ajuda no seguinte problema:

 Os inteiros positivos 1, 2, ..., n são colocados nos vértices de um
 n-ágono. Cada vértice é pintado de:

 *Vermelho, se ambos os números nos vértices vizinhos são maiores do que o
 número neste vértice;
 *Azul, se ambos os números nos vértices vizinhos são menores do que o
 número neste vértice;
 *Branco, se nenhuma das duas condições acima for satisfeita.
 Prove que o número de vértices vermelhos é igual ao número de vértices
 azuis.

 Também gostaria de algum material bom sobre contagem dupla, não
 necessariamente em português, caso alguém conhecesse.


Imagine que saiam flechas dos vértices menores para seus vizinhos maiores
(em vez de uma reta).
Para cada permutação, todo lado possuirá uma flecha correspondente (não
existem dois números iguais).

É possível fazer uma indução. A base é a disposição de flechas num mesmo
sentido (sentido horário por exemplo), nesta situação o número de vértices
de onde partem duas flechas (vértices azuis) é igual ao número de vértices
para onde chegam duas flechas (vértices vermelhos), note que esta base não
representa nenhuma disposição possível de se chegar com os dados do
problema:
a - b - c - d - ... - z - a

O passo de indução seria mudar uma única flecha de sentido e ver que os
números de vértices azuis e vermelhos continuam iguais.


Re: [obm-l] Problema

2009-09-22 Por tôpico Lucas Prado Melo
2009/9/22 Luís Eduardo Háteras luiseduardo...@hotmail.com

  Sou novo nessa lista e estou com dúvida nesse exercício, alguém saberia
 resolver ? E alguém sabe como me explicar porque não consegui compreender
 como resolver.

Primeira coisa tempo = comprimento / velocidade.
É preciso igualar o tempo que o trem leva para ultrapassar cada um e o tempo
que cada leva para chegar nas posições.

Por exemplo, o trem percorre um comprimento de C-30 (onde C é seu próprio
comprimento) para ultrapassar Bruno e Bruno percorre 30m neste mesmo tempo,
então:
(C-30)/Vt = 30/Vp

Onde Vt é velocidade do trem e Vp é velocidade das pessoas.

Basta encontrar a outra equação e resolver.


Re: [obm-l] OFF TOPIC

2009-09-08 Por tôpico Lucas Prado Melo
2009/9/1 staib st...@aman.ensino.eb.br

  Sei que alguns se incomodam quando usamos esse meio para ajudas que não
 se referem a olimpíadas matemáticas, perdoem-me.

Esta lista foi feita para discutir a Olimpíada Brasileira de Matemática, por
isso que o ideal é que se mantenha discutindo este assunto. Não é nada
pessoal.

 Estou precisando de artigos que se referem a Geometria Hiperbólica ou
 artigos que mostre  determinados teoremas da geometria euclidiana só são
 válidos por causa do quinto postulado.

Eu entendo que você queira pedir isto aqui, já que provavelmente existem
pesquisadores da área inscritos, mas neste caso específico, eu acredito que
existem muitas alternativas. Você poderia, por exemplo, procurar
pesquisadores na plataforma Lattes do CNPQ que trabalham com o assunto e
perguntar-lhes diretamente. Ou usar diversas ferramentas de busca (a CAPES
possui o portal de periódicos que ajuda a identificar textos de diversas
áreas, dependendo da organização a que vc está afiliado talvez você possa
até acessar os periódicos que a CAPES assina).

A SBM tem alguma lista de discussão para as diversas áreas de matemática?
(Não olhe pra mim, eu faço Ciência da Computação, e a SBC tem estas listas,
com livre acesso a não-associados). Talvez seja uma boa idéia criar estas
listas para divulgação de eventos e para os membros discutirem os assuntos
em que trabalham.

abraços

Ps: eu não sou moderador. ;-)


Re: [obm-l] Conjuntos

2009-01-21 Por tôpico Lucas Prado Melo
2009/1/21 Arthur Matta Moura art_mo...@hotmail.com:

  Quero saber se o 0 pertence ou não pertence aos Naturais, e por que não é
 definido a idéia de ordem para os Complexos.

O zero pertencer ou não aos naturais é mera questão técnica e os dois
casos são aceitos (cada autor tem o seu preferido e um ou outro se
torna mais conveniente conforme o assunto tratado). Isso acontece
porque muitos conceitos da matemática são criados como uma
racionalização posterior do que foi determinado antes (a axiomática de
Peano que define os números naturais, por exemplo, foi criada bem
depois de termos pensado a noção de número natural).
Os números complexos podem ter ordem definida, mas até agora nenhuma
ordem parece ser a mais natural (ou seja, uma ordem sensata que
fosse análoga à ordem usual para os naturais: 0  1  2  ... ).
Poderiamos tentar estabelecer que um número complexo Z seria menor que
outro complexo X se e somente se o módulo de Z for menor que o módulo
de X, mas isso não varia sentido por que i, 1, -i e -1 seriam tratados
como o mesmo número!

=
Instruções para entrar na lista, sair da lista e usar a lista em
http://www.mat.puc-rio.br/~obmlistas/obm-l.html
=


Re: [obm-l] formalização

2009-01-17 Por tôpico Lucas Prado Melo
On Sat, Jan 17, 2009 at 8:17 PM, Murilo Krell murilo.kr...@gmail.com wrote:
 Pessoal,
 numa prova de análise, para eu no meio da questão por exemplo, considerar
 lim (logn) - +00
 posso justificar isso de que forma?
 bastaria eu dizer que a função log é crescente?
Não basta dizer que é crescente...

1 - 1/(2^n) também é crescente (estritamente crescente) e tende a 1
com 'n' tendendo ao infinito.
Não sei muito de análise, posso argumentar que para todo elemento x da
sequência existe um elemento da sequência igual a x+1 e por isso a
sequência tende ao infinito (já que a existência de uma barreira
superior levaria a uma contradição)?

=
Instruções para entrar na lista, sair da lista e usar a lista em
http://www.mat.puc-rio.br/~obmlistas/obm-l.html
=


[obm-l] representação de pares ordenados

2009-01-05 Por tôpico Lucas Prado Melo
alguém conhece uma boa representação de par ordenado usando conjuntos?

=
Instruções para entrar na lista, sair da lista e usar a lista em
http://www.mat.puc-rio.br/~obmlistas/obm-l.html
=


Re: [obm-l] Combinatoria e Prob

2008-10-16 Por tôpico Lucas Prado Melo
2008/10/16 Thais Oliveira [EMAIL PROTECTED]:
 Olá pessoal, tudo bem?

 Nao consigo resolver dois exercicios de combinatoria.

 1) (FUVEST-1997) Os trabalhos da diretoria de um clube são realizados por
 seis comissões. Cada diretor participa exatamente de duas comissões e cada
 duas comissões têm exatamente um diretor comum.
 a) Quantos diretores tem o clube?
Se cada duas comissões definem um diretor (único), então o número de
diretores é igual à combinação 2 a 2 de comissões = 15.

 b) Escolhendo-se, ao acaso, dois diretores, qual é a probabilidade de que
 eles sejam de uma mesma comissão?
suponha que tenhamos um diretor já definido, então 2 comissões estão
selecionadas.
Agora vamos pegar as comissões do segundo diretor.
As chances de pegarmos de primeira uma comissão selecionada é de 2/6
se pegarmos uma não selecionada de primeira (chances de 4/6), as
chances de pegarmos uma selecionada é de 2/5: assim temos que as
chances desse segundo caso acontecer são de 4*2/(5*6) = 4/15
Somando tudo:  2/6 + 4/15 = (10+8)/30=18/30=9/15=3/5

=
Instruções para entrar na lista, sair da lista e usar a lista em
http://www.mat.puc-rio.br/~obmlistas/obm-l.html
=


Re: [obm-l] Combinatoria e Prob

2008-10-16 Por tôpico Lucas Prado Melo
2008/10/16 Thais Oliveira [EMAIL PROTECTED]:
 2) Uma recepcionista recebeu n chapéus, mas estes ficaram totalmente
 misturados. Decidiu, então, devolvê-los a esmo. Calcular a probabilidade de
 que nenhum homem receba seu chapéu.

Para essa questão, eu encontrei algo assim: (n! - (n-1)! + (n-2)! -
... (+-) 1) / n!
Mas não vou entrar nos detalhes disso pq suspeito que a resposta possa
estar errada...
Alguém tem alguma sugestão?

=
Instruções para entrar na lista, sair da lista e usar a lista em
http://www.mat.puc-rio.br/~obmlistas/obm-l.html
=


Re: [obm-l] Combinatoria e Prob

2008-10-16 Por tôpico Lucas Prado Melo
2008/10/16 Pedro Cardoso [EMAIL PROTECTED]:
 Eu supus que no sorteio o próprio José não pode ser sorteado.
E supos certo... acho que a falha nos meus argumentos é justamente não
ter suposto o mesmo.

[]'s

=
Instruções para entrar na lista, sair da lista e usar a lista em
http://www.mat.puc-rio.br/~obmlistas/obm-l.html
=


Re: [obm-l] SENHOR FONDI

2008-10-14 Por tôpico Lucas Prado Melo
2008/10/14 Lucas Prado Melo [EMAIL PROTECTED]:
 e também podemos estabelecer um limite inferior para q4:
 q531 = q4+q4=5131+q4 = q420
q4+q5=5131+q4 ...

 Voltando à eq do início: 4(q1+q2+q3+q4+q5) = 4(20 + q3 + 51) = 322+r
 = 4q3-28 = r
 Se calcularmos 4q3-28 para todas as possibilidades de q3, obtemos:
4q3-38

=
Instruções para entrar na lista, sair da lista e usar a lista em
http://www.mat.puc-rio.br/~obmlistas/obm-l.html
=


Re: [obm-l] SENHOR FONDI

2008-10-14 Por tôpico Lucas Prado Melo
2008/10/14 Denisson [EMAIL PROTECTED]:
 Tá faltando uma medida.
Eu supus que havia dois pares de números com a mesma soma...

=
Instruções para entrar na lista, sair da lista e usar a lista em
http://www.mat.puc-rio.br/~obmlistas/obm-l.html
=


Re: [obm-l] SENHOR FONDI

2008-10-14 Por tôpico Lucas Prado Melo
Oi,

minha resposta é 28kg:

considere que os pesos são q1q2q3q4q531
se somarmos todas as somas, temos:
[4q1 + (q2+q3+q4+q5)] + [3q2 + (q3+q4+q5)] + [2q3 + (q4+q5)] + [q4 +
(q5)] = 4(q1+q2+q3+q4+q5)
Sabemos que o número de somas é 10, mas apenas existem 9 somas únicas.
Assim, existe uma soma 'r' que é repetida, então: 4(q1+q2+q3+q4+q5) =
20+24+30+35+36+40+41+45+51+r = 322+r

q1+q2 é a menor soma = 20
q4+q5 é a maior soma = 51

Pode-se argumentar que a segunda maior soma precisa ser q3+q5
(q3+q4q3+q5!). Assim q3+q5=45 e q4-q3=6
como q5q4, q5  (q5+q4)/2  q4:
q5  51/2  q4
25=q4 (um limite superior para q4)

e também podemos estabelecer um limite inferior para q4:
q531 = q4+q4=5131+q4 = q420
Ou seja:
25=q4=21
e, dessa forma, 19=q3=15

Voltando à eq do início: 4(q1+q2+q3+q4+q5) = 4(20 + q3 + 51) = 322+r
= 4q3-28 = r
Se calcularmos 4q3-28 para todas as possibilidades de q3, obtemos:
22, 26, 30, 34 e 38
mas somente 30 é uma soma válida, assim q3 = 17, q4 = 23 e:
q5 + q4 = q5 + 23 = 51 = q5 = 28

[]'s

=
Instruções para entrar na lista, sair da lista e usar a lista em
http://www.mat.puc-rio.br/~obmlistas/obm-l.html
=


Re: [obm-l] SENHOR FONDI

2008-10-14 Por tôpico Lucas Prado Melo
On Tue, Oct 14, 2008 at 12:15 PM, *Vidal [EMAIL PROTECTED] wrote:
 Caro Arkon e Lucas,

 Dá para diminuir um pouquinho:

 S = q1 + q2 + q3 + q4 + q5
 4S = 322 + r

 Logo 322 + r é múltiplo de 4.
 A única possibilidade é r = 30.

Haha... boa.
Eu acho que o curso de ciência da computação me trouxe o hábito de
reduzir o número de coisas a serem testadas...

=
Instruções para entrar na lista, sair da lista e usar a lista em
http://www.mat.puc-rio.br/~obmlistas/obm-l.html
=


[obm-l] Limite para o infinito

2008-07-15 Por tôpico Lucas Prado Melo
Olá,

gostaria de saber como calcular limites tendendo ao infinito de
expressões da seguinte forma:
(a + 10^-b)^n - a^n
Com 'a' e 'b' naturais diferentes de 0 e 'n' tendendo ao infinito

[]'s

=
Instruções para entrar na lista, sair da lista e usar a lista em
http://www.mat.puc-rio.br/~obmlistas/obm-l.html
=


Re: [obm-l] Limite para o infinito

2008-07-15 Por tôpico Lucas Prado Melo
2008/7/15 Bruno França dos Reis [EMAIL PROTECTED]:
 De maneira geral, seja f(x) = b^n - a^n.
 Se a  b, f(x) -- +oo para x -- +oo.
 Se a = b, f(x) -- 0 para x -- +oo.
 se a  b, f(x) -- -oo para x -- -oo.
Obrigado!

E essa outra?
(a+10^-n)^n - a^n
Para 'a' natural diferente de 0 e 'n' tendendo ao infinito.

=
Instruções para entrar na lista, sair da lista e usar a lista em
http://www.mat.puc-rio.br/~obmlistas/obm-l.html
=


Re: [obm-l] Paradoxal...

2008-07-13 Por tôpico Lucas Prado Melo
2008/7/11 Maurício Collares [EMAIL PROTECTED]:
 Somas infinitas são definidas rigorosamente como o limite dos somas
 finitas quando o número de termos tende ao infinito (usando a
 definição com epsilons e deltas de tende ao infinito). A soma
 mencionada não existe porque a sequência das somas parciais dada (1,
 0, 1, 0, ...) tem duas subsequências que convergem para pontos
 diferentes. Ao colocar parênteses, você está na verdade tomando uma
 subsequência da sequência das somas parciais dada, que pode agora ter
 limite bem definido. Isto não implica, no entanto, que o limite
 original exista.
Ou seja, a associatividade não vale para somas infinitas?

=
Instruções para entrar na lista, sair da lista e usar a lista em
http://www.mat.puc-rio.br/~obmlistas/obm-l.html
=


Re: [obm-l] Composição de Funções

2007-12-29 Por tôpico Lucas Prado Melo
Se tivermos duas funções: f e g. Tal que  f: A-B   e g: B-C
então fog:A-C
Logo, no caso, acho que fof:R-{2}-R
O que anularia a questão. Alguém discorda?

On Dec 25, 2007 11:20 AM, Tales Prates Correia [EMAIL PROTECTED] wrote:

Olá !

   A função f, como você bem disse, tem como domínio mais extenso o
 conjunto D = R - {2}. Uma vez que a sua lei de correspondência é dada por


   f(x) = (2x + 1)/(x - 2)

   a lei de fof dá-se por

   (fof)(x) = x(x - 2)/(x - 2)

   E seu domínio ainda é D = R - {2}. Essa função não está definida
 no ponto x = 2 e é identica a função g real definida por g(x) = x no
 conjunto D.

   Por esse motivo, o exercício não apresenta alternativa correta.
 Desconsiderando esse erro dos formuladores, seria sensato apontar como
 correta

   a alternativa C.

   Espero ter ajudado.

   Abraços!

 
 From: [EMAIL PROTECTED]
 To: obm-l@mat.puc-rio.br
 Subject: [obm-l] Composição de Funções
 Date: Tue, 25 Dec 2007 01:36:03 -0300




 Olá,


 Dada a função f(x) = (2x + 1)/(x - 2), o valor se fof(2), é:

 a) 1/4 b) 1/2c) 2d) 4  e) -4


 O problema acima foi aplicado num exame de vestibular e o gabarito oficial
 apontava a alternativa C como correta.

 É fácil ver que f(2) não existe, pois D(f) = R - {2}. Desse modo, seríamos
 levados a concluir que o problema não apresenta alternativa correta, pois
 f(f(2)) não existe.

 Por outro lado, f(f(x)) = x, donde f(f(2)) = 2. O que parece ter sido levado
 em consideração pelo examinador.

 Pergunta: Onde está o erro?

 Abraços.




 
 Receba GRÁTIS as mensagens do Messenger no seu celular quando você estiver
 offline. Conheça o MSN Mobile! Crie já o seu!


=
Instruções para entrar na lista, sair da lista e usar a lista em
http://www.mat.puc-rio.br/~obmlistas/obm-l.html
=


Re: [obm-l] Filhos semelhantes...

2007-12-25 Por tôpico Lucas Prado Melo
E tem mais um complicador:o crossing-over.
Às vezes, dois cromossomos trocam algumas partes...

On Dec 19, 2007 8:28 PM, Ojesed Mirror [EMAIL PROTECTED] wrote:


 Ruy, está errado, o correto seria (1/2)^46, pois são 23 cromossomos do óvulo
 e 23 do espermatozoide.
 O 1/2 vem do fato da mitose resultar sempre duas células a partir de uma.

 Ojesed.

=
Instruções para entrar na lista, sair da lista e usar a lista em
http://www.mat.puc-rio.br/~obmlistas/obm-l.html
=


Re: [obm-l] Demonstrações

2007-12-23 Por tôpico Lucas Prado Melo
On Dec 16, 2007 11:56 PM, Sérgio Martins da Silva [EMAIL PROTECTED] wrote:
 Doutores,

 Penso que a palavra mais comum nesta lista e, quiçá, da matemática é
 demonstração. Por isto, gostaria de saber como se demonstra que uma
 demonstração está correta. E mais, que é completa. Quais são os requisitos,
 condições, etc ?

 Abraços,

 Sérgio

Oi,
Se eu estiver errado, por favor me corrijam,
Demonstrar que uma demonstração é válida é provar que a conclusão
deriva das premissas. (isso é lógica matemática)
Se, ao analisarmos uma prova a partir de suas premissas, chegamos (por
implicações sempre verdadeiras (também chamadas tautológicas)) à mesma
conclusão que a prova chegou, então a prova é válida, caso contrário
não.
Uma prova é dita completa quando não existem axiomas não declarados
(se eu não me engano).
Ex:
Se Alberto viajar e Bruno ir à praia
Então Daniel vai ao mercado
Prova:
Sabemos isso também:
- Se Alberto vai viajar e Bruno ir à praia, então Creuza vai limpar a
casa de Alberto
- Se Daniel não vai ao mercado, então Creuza não vai limpar a casa de
Alberto ou Alberto não vai viajar
Por lógica matemática:
A := Alberto ir viajar
B := Bruno ir à praia
C := Creuza ir limpar a casa de Alberto
D := Daniel ir ao mercado
Temos:
A e B e ( A e B - C ) e ( ¬D - ¬C ou ¬A )
Usando algumas regras de lógica:
( ¬D - ¬C ou ¬A ) = ( A e C - D )
A e B e ( A e B - C ) = C
A e C e ( A e C - D ) = D
Ou seja, D é verdade...

Resumindo (para não-leigos): uma prova é válida sse a conjunção das
premissas implica a conclusão da prova.

=
Instruções para entrar na lista, sair da lista e usar a lista em
http://www.mat.puc-rio.br/~obmlistas/obm-l.html
=


Re: [obm-l] Saida Lateral

2007-07-10 Por tôpico Lucas Prado Melo

É falsa, se M = 2, então temos (2*2-1)/3 = 1
e então continua 1, 1, 1, 1 ... indefinidamente

Em 10/07/07, Paulo Santa Rita[EMAIL PROTECTED] escreveu:

Ola Pessoal !

Considerem a seguinte questao :

A questao seguinte e interessante :seja M um natural impar maior que 1
e NAO DIVISIVEL por 3. A partir deste M vamos construir a seguinte
sequencia :

A1 = M

An+1 = ( (4*An) - 1 ) / 3 se An==1(MOD 3)
An+1 = ( (2*An) - 1 ) / 3 se An==2(MOD 3)

Se para algum n surgir An==0(MOD 3) a sequencia termina.

Eu afirmo que qualquer que seja o M de partida a sequencia sempre
termina. Esta minha afirmacao e verdadeira ou falsa ?

OBS : usei == para significar E CONGRUO A

Um Abracao a Todos
Paulo Santa Rita
3,1604,101007
=
Instruções para entrar na lista, sair da lista e usar a lista em
http://www.mat.puc-rio.br/~nicolau/olimp/obm-l.html
=



=
Instruções para entrar na lista, sair da lista e usar a lista em
http://www.mat.puc-rio.br/~nicolau/olimp/obm-l.html
=


Re: [obm-l] Saida Lateral

2007-07-10 Por tôpico Lucas Prado Melo

vcs da lista já repararam que eu não paro pra ler direito? ¬¬

Em 10/07/07, Lucas Prado Melo[EMAIL PROTECTED] escreveu:

É falsa, se M = 2, então temos (2*2-1)/3 = 1
e então continua 1, 1, 1, 1 ... indefinidamente

Em 10/07/07, Paulo Santa Rita[EMAIL PROTECTED] escreveu:
 Ola Pessoal !

 Considerem a seguinte questao :

 A questao seguinte e interessante :seja M um natural impar maior que 1
 e NAO DIVISIVEL por 3. A partir deste M vamos construir a seguinte
 sequencia :

 A1 = M

 An+1 = ( (4*An) - 1 ) / 3 se An==1(MOD 3)
 An+1 = ( (2*An) - 1 ) / 3 se An==2(MOD 3)

 Se para algum n surgir An==0(MOD 3) a sequencia termina.

 Eu afirmo que qualquer que seja o M de partida a sequencia sempre
 termina. Esta minha afirmacao e verdadeira ou falsa ?

 OBS : usei == para significar E CONGRUO A

 Um Abracao a Todos
 Paulo Santa Rita
 3,1604,101007
 =
 Instruções para entrar na lista, sair da lista e usar a lista em
 http://www.mat.puc-rio.br/~nicolau/olimp/obm-l.html
 =




=
Instruções para entrar na lista, sair da lista e usar a lista em
http://www.mat.puc-rio.br/~nicolau/olimp/obm-l.html
=


Re: [obm-l] Re: [obm-l] Questão Da OBM Nível 3

2007-07-07 Por tôpico Lucas Prado Melo

Explicando os valores:
Se temos um número de 4 dígitos, então o primeiro algarismo não pode
ser 0, restando 9 possibilidades para o primeiro algarismo (1, 2, 3,
4, 5, 6, 7, 8 e 9). O segundo, o terceiro e quarto algarismo podem ser
qualquer número de 0 a 9, ou seja 10 possibilidades
assim
9x10x10x10 = a quantidade de números de quatro dígitos

Vamos então calcular a quantidade de números não-peroba
Primeiro caso:
O número não-peroba é escrito no formato IPIP (ímpar, par, ímpar,
par; como 3214);
  O primeiro algarismo pode ser qualquer ímpar (1, 3, 5, 7, 9): 5
possibilidades
  O segundo algarismo pode ser qualquer par (0,2,4,6,8): 5 possibilidades
  O terceiro algarismo pode ser qualquer ímpar (1, 3, 5, 7, 9): 5
possibilidades
  O quarto algarismo pode ser qualquer par (0,2,4,6,8): 5 possibilidades
São 5x5x5x5 números não-peroba do primeiro caso.
Segundo caso:
  O número não peroba é escrito no formato PIPI (par, ímpar, par,
ímpar, como 2341)
O primeiro algarismo pode ser qualquer par exceto o zero
(2,4,6,8): 4 possibilidades
O segundo algarismo pode ser qualquer ímpar (1, 3, 5, 7, 9): 5
possibilidades
O terceiro algarismo pode ser qualquer par (0,2,4,6,8): 5 possibilidades
O quarto algarismo pode ser qualquer ímpar (1, 3, 5, 7, 9): 5
possibilidades
 São 4x5x5x5 números não-peroba do segundo caso.

[]'s

Em 06/07/07, Rodolfo Braz[EMAIL PROTECTED] escreveu:

Oi Rafael primeiramente muito obrigado por está me ajudando! Cara me explica
mais detalhadamente de onde veio os 9000 e os outros dois valores? Abraço!

rgc [EMAIL PROTECTED] escreveu:

Oi
Eu pensei assim, veja se da pra entender:
Só existem dois tipos de números de 4 digitos nesse problema: os perobas e
os não perobas. O jeito mais simples é contar quantos números de 4 digitos
existem, depois tirar os não perobas. Há 9*10*10*10=9000 números de 4
algarismos. Para que um número não seja peroba deve ter todos os digitos
vizinhos com paridade diferente. Representando par=P e ímpar=I, se o
primeiro digito é P o segundo é I, o 3° é P e o 4° é I. Lembrando que o
primeiro não pode ser zero há 4*5*5*5=500. Se o 1° digito é I, o segundo é
P, o 3° é I e o quarto é P. Logo há 5*5*5*5=625. Somando achamos que os não
perobas são 1125. Então os perobas são 9000-1125=7875.

- Original Message -
From: Rodolfo Braz
To: Lista De Discussão OBM
Sent: Friday, July 06, 2007 11:53 AM
Subject: [obm-l] Questão Da OBM Nível 3

Pessoal gostaria se possível que alguém solucionasse essa questão
detalhadamente para mim por favor pois não consigo entender a solução
proposta pelo pessoal da OBM. Desde já fico muito grato!

Um número de quatro dígitos é dito peroba se possui pelo menos dois dígitos
vizinhos com a mesma paridade. Quantos números perobas existem?
A) 8999 B) 8874 C) 7875 D) 8000 E) 7750



 
 Novo Yahoo! Cadê? - Experimente uma nova busca.




 
Novo Yahoo! Cadê? - Experimente uma nova busca.




=
Instruções para entrar na lista, sair da lista e usar a lista em
http://www.mat.puc-rio.br/~nicolau/olimp/obm-l.html
=


Re: [obm-l] Problema de Desigualdade

2007-05-07 Por tôpico Lucas Prado Melo

Eu não entendi isso:
tgA tgB + tgA tgC + tgB tgC = 1 - A+B+C = Pi/2
Poderia esclarer para mim, por favor?

Em 06/05/07, charles[EMAIL PROTECTED] escreveu:


Sejam x, y, z reais positivos tais que xy + yz + zx = 1. Prove que:
2x (1 - x²) +  2y (1 - y²) + 2z (1 - z²)   x+ y+z

  (1+x²)²  (1+y²)² (1+z²)² 1+x² 1+y²  1+z²

 De a função tangente ser bijetora no intervalo [0,pi/2], nos reais
positivos, existe apenas um tgA=x, tgB=y e TgC=z.
 Da formula da soma da tangente de trs termos(calcule) temos que tgATgB
+TgBTgC +TgATgC=1 se e só se A + B + C= pi/2 aí de (tga)^2 +1=(secx)^2 e de
1-(tgx)^2=cos(2x)/(cosx)^2, a des fica:

depois fica um negocio assim: sen(4A) + sen(4B)+sen(4C) = sen(2A)
+sen(2B)+sen(2C)


  lado direito:
2sen(a+b)cos(a-b)+sen(2(a+b))=2sen(a+b)[cos(a-b)+cos(a+b)]=4sen(a+b)[cosacosb]

 lado esquerdo:
2sen(2a+2b)cos(2a-2b)-2sen[2a+2B]cos(2a+2b)=2sen(2a+2b)[cos(2a-2b)-cos(2a+2b)]=4sena+bcos(a+b)2sen(2a)sen(2b)
  fica,


cancelando: sena.senb.senc=1/8 que sai por JENSEN, senx é concava em
0,pi/2 derivada segunda é menor que zero (-cosx, x pertencente a 0,pi/2)
logo:
   sena.senb.senc=(sena+senb+senc)^3/3^3 (média aritmética-geométrica)
=(3.sen(pi/6))^3/3^3(des.jensen)=1/8, pronto.
provavelmente tem algum erro ae mas ali no meio da parte de
trigonometria eu usei as fórmulas de tranformar soma em produto algumas
vezes. Mas cara na tua escola voce tem um professor só para obm? eu queria
estudar numa escola assim.voce tem sorte!



 Obrigado!


 __
 Fale com seus amigos de graça com o novo Yahoo! Messenger
 http://br.messenger.yahoo.com/




=
Instruções para entrar na lista, sair da lista e usar a lista em
http://www.mat.puc-rio.br/~nicolau/olimp/obm-l.html
=


[obm-l] somatório dos inversos dos naturais

2007-05-04 Por tôpico Lucas Prado Melo

Existe algum modo de expressar a soma 1/1 + 1/2 + 1/3 + ... + 1/n em
função de 'n'?

=
Instruções para entrar na lista, sair da lista e usar a lista em
http://www.mat.puc-rio.br/~nicolau/olimp/obm-l.html
=


Re: [obm-l] Re: [obm-l] Re:[obm-l] Teoria dos números

2007-04-28 Por tôpico Lucas Prado Melo

hum... li errado Oo
Desculpe...

Em 28/04/07, Lucas Prado Melo[EMAIL PROTECTED] escreveu:


   Amigos, ajude-me nessas questões:

 1) Ache o menor número natural terminado em 56, divisível por 56, e com a
soma dos seus algarismos igual a 56.

Ok ...
Temos que   56 k == 56 (mod 100) = 56k - 56 == 0 (mod 100) =  56(k-1)  ==
0  (mod 100)
ok, 56  = 7 x 2^3, para 100 dividir 56(k-1) = 2^3x7x(k-1), 25 precisa
dividir k-1. O menor caso é quando k-1 é igual a 25, logo k = 26.

Nosso número, portanto é 56x26 = 1456



=
Instruções para entrar na lista, sair da lista e usar a lista em
http://www.mat.puc-rio.br/~nicolau/olimp/obm-l.html
=


Re: [obm-l] Re: [obm-l] Re:[obm-l] Teoria dos números

2007-04-28 Por tôpico Lucas Prado Melo

   Amigos, ajude-me nessas questões:


1) Ache o menor número natural terminado em 56, divisível por 56, e com a

soma dos seus algarismos igual a 56.

Ok ...
Temos que   56 k == 56 (mod 100) = 56k - 56 == 0 (mod 100) =  56(k-1)  ==
0  (mod 100)
ok, 56  = 7 x 2^3, para 100 dividir 56(k-1) = 2^3x7x(k-1), 25 precisa
dividir k-1. O menor caso é quando k-1 é igual a 25, logo k = 26.

Nosso número, portanto é 56x26 = 1456


Re: [obm-l] Função inversa

2007-04-22 Por tôpico Lucas Prado Melo

Existe alguma fonte que fale das equações transcendentais?

Em 22/04/07, Marcelo Salhab Brogliato[EMAIL PROTECTED] escreveu:

Ola Max,
ate onde sei esta eh uma equacao transcendental e sua inversa nao pode
ser obtida analiticamente.
Se quisermos determinar x, tal que: f(x) = c, temos que utilizar
metodos graficos ou numericos.

abracos,
Salhab

On 4/22/07, Max R. [EMAIL PROTECTED] wrote:


   Temos f(x) = x + e^x  , calcule a inversa de f (x)

   Agradeço desde já

 
 Ligue para os amigos com a Chamada de PC para PC - GRATUITO Experimente já!

=
Instruções para entrar na lista, sair da lista e usar a lista em
http://www.mat.puc-rio.br/~nicolau/olimp/obm-l.html
=



=
Instruções para entrar na lista, sair da lista e usar a lista em
http://www.mat.puc-rio.br/~nicolau/olimp/obm-l.html
=